[Boards: 3 / a / aco / adv / an / asp / b / bant / biz / c / can / cgl / ck / cm / co / cock / d / diy / e / fa / fap / fit / fitlit / g / gd / gif / h / hc / his / hm / hr / i / ic / int / jp / k / lgbt / lit / m / mlp / mlpol / mo / mtv / mu / n / news / o / out / outsoc / p / po / pol / qa / qst / r / r9k / s / s4s / sci / soc / sp / spa / t / tg / toy / trash / trv / tv / u / v / vg / vint / vip / vp / vr / w / wg / wsg / wsr / x / y ] [Search | Free Show | Home]

Anybody know how to solve this? I have been trying to solve this

This is a blue board which means that it's for everybody (Safe For Work content only). If you see any adult content, please report it.

Thread replies: 19
Thread images: 4

File: Screenshot (3).png (51KB, 767x340px) Image search: [Google]
Screenshot (3).png
51KB, 767x340px
Anybody know how to solve this? I have been trying to solve this for the past few hours ;_;
>>
Since it's symmetric, you can draw spokes coming out from the middle which divide the figure into 8 identical right triangles. You can just solve one triangle and then extrapolate the answer from there.

Let's consider the one triangle that is formed by drawing a line segment A from the middle point to the left, parallel to the bottom of the square, and the line segment B drawn from the middle point up and to the left so that it hits the top left corner of the square. There is a 45 degree angle between line A and B. We know line segment A has length 1, and B is length sqrt(2) from Pythagorean theorem.

The shaded region of this triangle can be calculated easily using trig because it bisects lines A and B, and we know the angle between them. Note that besides line A and B, the third boundary of this region must be a line because of the properties of the problem, so the shaded region is also a triangle.
>>
>>8727001
I tried that, but isn't the region not a triangle? It is curved, so doesn't that not apply?
>>
>>8726965
>besides line A and B, the third boundary of this region must be a line
whyy
shit doesn't look like a line to me, it all looks curved and there are no break points in the "sides" of the orange figure
>>
>>8727009
op, try defining a function that is 1 when a point in the square is closer to the center and 0 when it's farther away
use it to "count" the points by integrating somehow

can a mathfag confirm or deny if this can be done
>>
The boundary nearest to a side side would be the set of points equidistant from the center and that side.

This is one of the classic definitions of a parabola:
https://en.wikipedia.org/wiki/Parabola#Definition_of_a_parabola_as_locus_of_points
>>
>>8727021
i knew it, literally a square distance function
so i'm basically right here>>8727019
>>
Ahhhhh, I see. We gotta make the two points equidistant. Thus x^2+y^2=(1-y)^2, where x and y are the coordinates of the region. Basically, gotta make the hypotenuses of the triangle equal to the distance of the region to the side of the square
>>
Have fun.
>>
>>8727112
Messed up the theta limits. It's pi/4 not pi/2
>>
>>8727112
I thought it would be easier in polar coordinates. It's not. I don't think I messed up this time.
>>
>>8727160
I tried that too. Best use what >>8727001 said. Didn't seem intuitive at first, but makes sense now
>>
>>8727192
>Didn't seem intuitive at first, but makes sense now

That's because your initial drawing is plain wrong. Whoever gave this to you was trolling you hard.
>>
>>8727235
fucking TAs my dude. God damn it
>>
>>8727254
Fuck no.
That guy >>8727001 is wrong. I guess the drawing is right.
>>
File: Sans titre.png (73KB, 800x600px) Image search: [Google]
Sans titre.png
73KB, 800x600px
>>8727254
>>
>>8726965
the shaded area is a square of radius 1
>>
>>8727291
Ok it's done. It was fucking easy in retrospect. The answer (for a square of side 2) is 8/3

Have you guys done any measure theory ?


[math] \displaystyle \int_0^1 \int_0^1 1_{y \leq \frac{1-x^2}{2}} dx dy [/math]
>>
>>8727320
whoops went full faggot. I integrated over a full quadrant, not the half of one.
The result should be something like

Integrating colored part in the top-rtight part :
[math] \displaystyle \int_0^1 \int_0^1 1_{y \leq \frac{1-x^2}{2}} 1_{y \geq x} dx dy = \displaystyle \int_0^1 (\frac{1-x^2}{2} -x) 1_{\frac{1-x^2}{2} \geq x} dx [/math]

The final answer would be [math]\frac{4}{3}(4\sqrt{2} -5) \simeq 0.9[/math] which seems more reasonable.
Thread posts: 19
Thread images: 4


[Boards: 3 / a / aco / adv / an / asp / b / bant / biz / c / can / cgl / ck / cm / co / cock / d / diy / e / fa / fap / fit / fitlit / g / gd / gif / h / hc / his / hm / hr / i / ic / int / jp / k / lgbt / lit / m / mlp / mlpol / mo / mtv / mu / n / news / o / out / outsoc / p / po / pol / qa / qst / r / r9k / s / s4s / sci / soc / sp / spa / t / tg / toy / trash / trv / tv / u / v / vg / vint / vip / vp / vr / w / wg / wsg / wsr / x / y] [Search | Top | Home]

I'm aware that Imgur.com will stop allowing adult images since 15th of May. I'm taking actions to backup as much data as possible.
Read more on this topic here - https://archived.moe/talk/thread/1694/


If you need a post removed click on it's [Report] button and follow the instruction.
DMCA Content Takedown via dmca.com
All images are hosted on imgur.com.
If you like this website please support us by donating with Bitcoins at 16mKtbZiwW52BLkibtCr8jUg2KVUMTxVQ5
All trademarks and copyrights on this page are owned by their respective parties.
Images uploaded are the responsibility of the Poster. Comments are owned by the Poster.
This is a 4chan archive - all of the content originated from that site.
This means that RandomArchive shows their content, archived.
If you need information for a Poster - contact them.